Eigenvalues and vectors of a 4 by 4 matrix

In summary: Sorry for the confusion. To solve this quartic equation, you can use a variety of methods such as the Rational Root Theorem or the Descartes' Rule of Signs to narrow down the possible rational roots. Once you have a few possible roots, you can use synthetic division or the quadratic formula to find the remaining roots. Alternatively, you can use a computer program or calculator to solve for the roots.
  • #1
Ron Burgundypants
23
0

Homework Statement



Coupled Harmonic Oscillators. In this series of exercises you are asked
to generalize the material on harmonic oscillators in Section 6.2 to the
case where the oscillators are coupled. Suppose there are two masses m1
and m2 attached to springs and walls as shown in Figure 6.10. The springs
connecting mj to the walls both have spring constants k1, while the spring
connecting m1 and m2 has spring constant k2. This coupling means that
the motion of either mass affects the behavior of the other.
Let xj denote the displacement of each mass from its rest position, and
assume that both masses are equal to 1. The differential equations for
these coupled oscillators are then given by

x1'' = -k(1 + k2)x1 + k2x2
x2'' = k2x1 - (k1 + k2)x2

These equations are derived as follows. If m1 is moved to the right
(x1 > 0), the left spring is stretched and exerts a restorative force on
m1 given by -k1x1. Meanwhile, the central spring is compressed, so it
exerts a restorative force on m1 given by -k2x1. If the right spring is
stretched, then the central spring is compressed and exerts a restorative
force on m1 given by k2x2 (since x2 < 0). The forces on m2 are similar.

(a) Write these equations as a first-order linear system.
(b) Determine the eigenvalues and eigenvectors of the corresponding
matrix.
(c) Find the general solution.
(d) Let ω1 = √k1 and ω2 = √k1 + 2k2. What can be said about the
periodicity of solutions relative to the ωj? Prove this.

Homework Equations



(A - λI) = Ax

The Attempt at a Solution



Part a is ok but I'm stuck for the rest

For part a we set y1 = dx1/dt and therefore dy1/dt = d2x/dt2
Then y2= dx2/dt so we have a similar case of dy2/dt = d2x/dt2

I think this is enough to satisfy part a) of the question

So then we put them into matrix form to try and find the eigenvalues and vectors and get the following by rows. Columns are x1,y1, x2, y2 respectively.

dx1/dt = 0 1 0 0
dy1/dt = -(k1+k2) 0 k2 0
dx2/dt = 0 0 0 1
dy2/dt = k2 0 -(k1+k2) 0

So there is a diagonal of zeros meaning we have zero trace. By using the formula (A - λI) = Ax. An attempt was made to try and get the eigenvalues. I subtract λ across each term in the diagonal, multiply out the expression and I end up with the following expression.

k12λ + k1k2λ + k22λ. I don't know how to deal with this expression. I'm used to seeing things in the form λ2 + λ - c.

So any helpful tips?
 
Physics news on Phys.org
  • #2
Ron Burgundypants said:

Homework Statement



Coupled Harmonic Oscillators. In this series of exercises you are asked
to generalize the material on harmonic oscillators in Section 6.2 to the
case where the oscillators are coupled. Suppose there are two masses m1
and m2 attached to springs and walls as shown in Figure 6.10. The springs
connecting mj to the walls both have spring constants k1, while the spring
connecting m1 and m2 has spring constant k2. This coupling means that
the motion of either mass affects the behavior of the other.
Let xj denote the displacement of each mass from its rest position, and
assume that both masses are equal to 1. The differential equations for
these coupled oscillators are then given by

x1'' = -k(1 + k2)x1 + k2x2
x2'' = k2x1 - (k1 + k2)x2

These equations are derived as follows. If m1 is moved to the right
(x1 > 0), the left spring is stretched and exerts a restorative force on
m1 given by -k1x1. Meanwhile, the central spring is compressed, so it
exerts a restorative force on m1 given by -k2x1. If the right spring is
stretched, then the central spring is compressed and exerts a restorative
force on m1 given by k2x2 (since x2 < 0). The forces on m2 are similar.

(a) Write these equations as a first-order linear system.
(b) Determine the eigenvalues and eigenvectors of the corresponding
matrix.
(c) Find the general solution.
(d) Let ω1 = √k1 and ω2 = √k1 + 2k2. What can be said about the
periodicity of solutions relative to the ωj? Prove this.

Homework Equations



(A - λI) = Ax

The Attempt at a Solution



Part a is ok but I'm stuck for the rest

For part a we set y1 = dx1/dt and therefore dy1/dt = d2x/dt2
Then y2= dx2/dt so we have a similar case of dy2/dt = d2x/dt2

I think this is enough to satisfy part a) of the question

So then we put them into matrix form to try and find the eigenvalues and vectors and get the following by rows. Columns are x1,y1, x2, y2 respectively.

dx1/dt = 0 1 0 0
dy1/dt = -(k1+k2) 0 k2 0
dx2/dt = 0 0 0 1
dy2/dt = k2 0 -(k1+k2) 0

So there is a diagonal of zeros meaning we have zero trace. By using the formula (A - λI) = Ax. An attempt was made to try and get the eigenvalues. I subtract λ across each term in the diagonal, multiply out the expression and I end up with the following expression.

k12λ + k1k2λ + k22λ. I don't know how to deal with this expression. I'm used to seeing things in the form λ2 + λ - c.

So any helpful tips?
Let's call your matrix A. Write the matrix ##A - \lambda I##, and then take its determinant, which you set equal to zero. That should give you an equation that you can solve for ##\lambda##.

The idea is that if ##\lambda## is an eigenvalue (with associated eigenvector ##\vec x##, then by definition, ##A\vec x = \lambda \vec x##. Equivalently, ##(A - \lambda I)\vec x = \vec 0##, which for nonzero vectors ##\vec x##, means that ##|A - \lambda I| = 0##.
 
  • #3
Ok maybe I wasn't clear enough. Thanks for your response. I'm aware this is how you find the eigenvalue. My question is how do i solve the final expression to give me some eigenvalues. I also just realized I made a mistake...

My actual final expression is λ4 - 2λ3k2 + λ2(k22+k2 - k1) + λ(k1k2 - k22) = 0

Getting eigenvalues form this? Yikes...
 
  • #4
Ron Burgundypants said:
Ok maybe I wasn't clear enough. Thanks for your response. I'm aware this is how you find the eigenvalue. My question is how do i solve the final expression to give me some eigenvalues. I also just realized I made a mistake...

My actual final expression is λ4 - 2λ3k2 + λ2(k22+k2 - k1) + λ(k1k2 - k22) = 0

Getting eigenvalues form this? Yikes...

The first eigenvalue is obvious; the second requires a bit of guesswork; and then you have a quadratic.
 

1. What are eigenvalues and eigenvectors?

Eigenvalues and eigenvectors are mathematical concepts used to analyze linear transformations on a vector space. Eigenvalues represent the scalar values by which an eigenvector is scaled when the transformation is applied, and eigenvectors are the vectors that remain in the same direction when the transformation is applied.

2. How do you find eigenvalues of a 4x4 matrix?

To find the eigenvalues of a 4x4 matrix, you first need to calculate the characteristic polynomial by subtracting the identity matrix multiplied by the eigenvalue from the original matrix. Then, solve the polynomial equation to find the eigenvalues.

3. Can a 4x4 matrix have complex eigenvalues?

Yes, a 4x4 matrix can have complex eigenvalues. The eigenvalues of a matrix do not have to be real numbers; they can be complex numbers as well.

4. What is the relationship between eigenvalues and determinants?

The determinant of a matrix is the product of its eigenvalues. This means that the determinant can be used to calculate the eigenvalues of a matrix, and vice versa.

5. How are eigenvectors and eigenvalues used in real-world applications?

Eigenvectors and eigenvalues have various applications in fields such as physics, engineering, and computer science. They can be used to analyze the stability of a system, find optimal solutions, and reduce the complexity of mathematical problems.

Similar threads

Replies
5
Views
361
  • Introductory Physics Homework Help
Replies
19
Views
2K
  • Programming and Computer Science
Replies
15
Views
2K
  • Introductory Physics Homework Help
Replies
2
Views
2K
  • Advanced Physics Homework Help
Replies
4
Views
4K
  • Introductory Physics Homework Help
Replies
2
Views
714
  • Programming and Computer Science
Replies
2
Views
3K
  • Advanced Physics Homework Help
Replies
1
Views
5K
  • Differential Equations
Replies
1
Views
4K
  • Programming and Computer Science
Replies
1
Views
2K
Back
Top